K
Khách

Hãy nhập câu hỏi của bạn vào đây, nếu là tài khoản VIP, bạn sẽ được ưu tiên trả lời.

25 tháng 4 2018

1 do (x-1)4 là số tự nhiên,(y+1)^4 là số tự nhiên 

nên để tổng bằng 0 thì cả (x-1)4 và (y+1)^4cùng bằng 0

nên x=0,y=-1

thay x,y vào rồi tính C

25 tháng 4 2018

ta có:\(A=\left|x+1\right|+\left|x+2\right|+...+\left|x+9\right|=14x\left(1\right)\)

do \(\left|x+1\right|\ge0,\left|x+2\right|\ge0,....,\left|x+9\right|\ge0\)

\(\Rightarrow14x>0\)\(\Rightarrow x>0\)

khi đó (1) trở thành:x+1+x+2+x+3+...+x+9=14x

\(\Rightarrow9x+45=14x\)

\(\Rightarrow45=5x\)

\(\Rightarrow x=9\)

24 tháng 12 2018

\(\Rightarrow3+\frac{y+z-2x}{x}=3+\frac{x+z-2y}{y}=3+\frac{x+y-2z}{z}\)

\(\Rightarrow\frac{x+y+z}{x}=\frac{x+y+z}{y}=\frac{x+y+z}{z}\)

\(TH1:x+y+z=0\)

\(\Rightarrow x=-\left(y+z\right),y=-\left(x+z\right),z=-\left(x+y\right)\)

\(A=\left(1+\frac{-y-z}{y}\right).\left(1+\frac{-x-z}{z}\right).\left(1+\frac{-x-y}{x}\right)\)

\(A=-\left(\frac{z}{y}\cdot\frac{x}{z}\cdot\frac{y}{x}\right)=-1\)

\(TH2:x+y+z\ne0\)

\(\Rightarrow x=y=z\Rightarrow A=2^3=8\)

sai đề ròi: tớ làm 2 trường hợp luôn vì trường hợp x+y+z khác 0 thì A mới t/m thuộc N 

mà đề là x+y+z khác 0 -.-

24 tháng 12 2018

cảm ơn nhiều

10 tháng 3 2023

vũ 7b hả

 

10 tháng 8 2017

post từng câu một thôi bn nhìn mệt quá

23 tháng 1 2021

1) Áp dụng bất đẳng thức AM - GM và bất đẳng thức Schwarz:

\(P=\dfrac{1}{a}+\dfrac{1}{\sqrt{ab}}\ge\dfrac{1}{a}+\dfrac{1}{\dfrac{a+b}{2}}\ge\dfrac{4}{a+\dfrac{a+b}{2}}=\dfrac{8}{3a+b}\ge8\).

Đẳng thức xảy ra khi a = b = \(\dfrac{1}{4}\).

NV
23 tháng 1 2021

2.

\(4=a^2+b^2\ge\dfrac{1}{2}\left(a+b\right)^2\Rightarrow a+b\le2\sqrt{2}\)

Đồng thời \(\left(a+b\right)^2\ge a^2+b^2\Rightarrow a+b\ge2\)

\(M\le\dfrac{\left(a+b\right)^2}{4\left(a+b+2\right)}=\dfrac{x^2}{4\left(x+2\right)}\) (với \(x=a+b\Rightarrow2\le x\le2\sqrt{2}\) )

\(M\le\dfrac{x^2}{4\left(x+2\right)}-\sqrt{2}+1+\sqrt{2}-1\)

\(M\le\dfrac{\left(2\sqrt{2}-x\right)\left(x+4-2\sqrt{2}\right)}{4\left(x+2\right)}+\sqrt{2}-1\le\sqrt{2}-1\)

Dấu "=" xảy ra khi \(x=2\sqrt{2}\) hay \(a=b=\sqrt{2}\)

3. Chia 2 vế giả thiết cho \(x^2y^2\)

\(\dfrac{1}{x}+\dfrac{1}{y}=\dfrac{1}{x^2}+\dfrac{1}{y^2}-\dfrac{1}{xy}\ge\dfrac{1}{4}\left(\dfrac{1}{x}+\dfrac{1}{y}\right)^2\)

\(\Rightarrow0\le\dfrac{1}{x}+\dfrac{1}{y}\le4\)

\(A=\left(\dfrac{1}{x}+\dfrac{1}{y}\right)\left(\dfrac{1}{x^2}+\dfrac{1}{y^2}-\dfrac{1}{xy}\right)=\left(\dfrac{1}{x}+\dfrac{1}{y}\right)^2\le16\)

Dấu "=" xảy ra khi \(x=y=\dfrac{1}{2}\)

NV
2 tháng 2

Pt đầu tương đương: \(\sqrt[3]{x^2}+2\sqrt[3]{y^2}+4\sqrt[3]{z^2}=7\)

Pt 2 tương đương:

\(\left(xy^2+z^4\right)^2-\left(xy^2-z^4\right)^2=4\)

\(\Leftrightarrow4xy^2z^4=4\)

\(\Leftrightarrow xy^2z^4=1\) (1)

Quay lại pt đầu, áp dụng AM-GM:

\(7=\sqrt[3]{x^2}+\sqrt[3]{y^2}+\sqrt[3]{y^2}+\sqrt[3]{z^2}+\sqrt[3]{z^2}+\sqrt[3]{z^2}+\sqrt[3]{z}\ge7\sqrt[7]{\sqrt[3]{x^2}.\sqrt[3]{y^4}.\sqrt[3]{z^8}}\)

\(\Leftrightarrow\sqrt[21]{x^2y^4z^8}\le1\)

\(\Leftrightarrow x^2y^4z^8\le1\)

\(\Rightarrow\left|xy^2z^4\right|\le1\Rightarrow xy^2z^4\le1\)

Dấu "=" xảy ra khi và chỉ khi:

\(\left\{{}\begin{matrix}x^2=y^2=z^2\\xy^2z^4=1\\x>0\end{matrix}\right.\) \(\Rightarrow\left\{{}\begin{matrix}x=1\\y=\pm1\\z=\pm1\end{matrix}\right.\)

Các bộ thỏa mãn là: \(\left(1;1;1\right);\left(1;1;-1\right);\left(1;-1;1\right);\left(1;-1;-1\right)\)

2 tháng 2

Anh ơi! Điều kiện x>0 là như nào ạ anh. 

25 tháng 10 2020

Bài 4: Áp dụng bất đẳng thức AM - GM, ta có: \(P=\text{​​}\Sigma_{cyc}a\sqrt{b^3+1}=\Sigma_{cyc}a\sqrt{\left(b+1\right)\left(b^2-b+1\right)}\le\Sigma_{cyc}a.\frac{\left(b+1\right)+\left(b^2-b+1\right)}{2}=\Sigma_{cyc}\frac{ab^2+2a}{2}=\frac{1}{2}\left(ab^2+bc^2+ca^2\right)+3\)Giả sử b là số nằm giữa a và c thì \(\left(b-a\right)\left(b-c\right)\le0\Rightarrow b^2+ac\le ab+bc\)\(\Leftrightarrow ab^2+bc^2+ca^2\le a^2b+abc+bc^2\le a^2b+2abc+bc^2=b\left(a+c\right)^2=b\left(3-b\right)^2\)

Ta sẽ chứng minh: \(b\left(3-b\right)^2\le4\)(*)

Thật vậy: (*)\(\Leftrightarrow\left(b-4\right)\left(b-1\right)^2\le0\)(đúng với mọi \(b\in[0;3]\))

Từ đó suy ra \(\frac{1}{2}\left(ab^2+bc^2+ca^2\right)+3\le\frac{1}{2}.4+3=5\)

Đẳng thức xảy ra khi a = 2; b = 1; c = 0 và các hoán vị

26 tháng 10 2020

Bài 1: Đặt \(a=xc,b=yc\left(x,y>0\right)\)thì điều kiện giả thiết trở thành \(\left(x+1\right)\left(y+1\right)=4\)

Khi đó  \(P=\frac{x}{y+3}+\frac{y}{x+3}+\frac{xy}{x+y}=\frac{x^2+y^2+3\left(x+y\right)}{xy+3\left(x+y\right)+9}+\frac{xy}{x+y}\)\(=\frac{\left(x+y\right)^2+3\left(x+y\right)-2xy}{xy+3\left(x+y\right)+9}+\frac{xy}{x+y}\)

Có: \(\left(x+1\right)\left(y+1\right)=4\Rightarrow xy=3-\left(x+y\right)\)

Đặt \(t=x+y\left(0< t< 3\right)\Rightarrow xy=3-t\le\frac{\left(x+y\right)^2}{4}=\frac{t^2}{4}\Rightarrow t\ge2\)(do t > 0)

Lúc đó \(P=\frac{t^2+3t-2\left(3-t\right)}{3-t+3t+9}+\frac{3-t}{t}=\frac{t}{2}+\frac{3}{t}-\frac{3}{2}\ge2\sqrt{\frac{t}{2}.\frac{3}{t}}-\frac{3}{2}=\sqrt{6}-\frac{3}{2}\)với \(2\le t< 3\)

Vậy \(MinP=\sqrt{6}-\frac{3}{2}\)đạt được khi \(t=\sqrt{6}\)hay (x; y) là nghiệm của hệ \(\hept{\begin{cases}x+y=\sqrt{6}\\xy=3-\sqrt{6}\end{cases}}\)

Ta lại có \(P=\frac{t^2-3t+6}{2t}=\frac{\left(t-2\right)\left(t-3\right)}{2t}+1\le1\)(do \(2\le t< 3\))

Vậy \(MaxP=1\)đạt được khi t = 2 hay x = y = 1

19 tháng 1 2021

Cách này đòi hỏi sự kiên nhẫn và kinh nghiệm.

Cần chứng minh:

\({\dfrac {4 \left( xy+zx+yz \right) \left( x+y+z \right) ^{7}}{ 243}}- \left( {x}^{3}+{y}^{3}+{z}^{3} \right) \left( {x}^{3}{y}^{3}+{ x}^{3}{z}^{3}+{y}^{3}{z}^{3} \right) \geqslant 0.\quad(1) \) 

Đặt 

\(\text{M}=4\,{z}^{7}+ \left( 757\,x+757\,y \right) {z}^{6}+84\, \left( x+y \right) ^{2}{z}^{5}+140\, \left( x+y \right) ^{3}{z}^{4}\\\quad\quad+ \left( 1598 \,{x}^{4}+4205\,{x}^{3}y+4971\,{x}^{2}{y}^{2}+4205\,x{y}^{3}+1598\,{y} ^{4} \right) {z}^{3}\\\quad \quad+84\, \left( x+y \right) ^{5}{z}^{2}+28\, \left( x +y \right) ^{6}z\geqslant 0 \)

Ta có:

\((1)\Leftrightarrow \dfrac{1}{243}xy\cdot M+{\dfrac { \left( x+y \right) \left( {x}^{2}+11\,xy+{y}^{2} \right) \left( 2\,x-y \right) ^{2} \left( x-2\,y \right) ^{2}xy}{243}}\\\quad\quad+{ \dfrac { \left( x+y \right) z \left( x+y+z \right) \left( {x}^{2}+2\,x y+11\,zx+{y}^{2}+11\,yz+{z}^{2} \right) \left( 2\,y-z+2\,x \right) ^{ 2} \left( y-2\,z+x \right) ^{2}}{243}}\geqslant 0. \)

Đẳng thức xảy ra khi $...$